Proving the One-to-One Property and Image of a Complex Function

Click For Summary
The discussion focuses on proving that the function f(z) = (1 - iz) / (1 + iz) is one-to-one and maps the unit disk D to the right half-plane. The user has successfully shown that f(z1) = f(z2) implies z1 = z2, confirming the one-to-one property. However, they are uncertain about demonstrating that the image of f(D) is the set of w where Re(w) > 0, particularly regarding the sign of the denominator when y > 0. They seek clarification on whether proving the image's inclusion is sufficient or if reverse inclusion is also necessary. The conversation highlights the complexities of complex analysis and the importance of understanding function behavior in different regions.
BSMSMSTMSPHD
Messages
131
Reaction score
0

Homework Statement



Let f(z) = \frac{1-iz}{1+iz} and let \mathbb{D} = \{z : |z| < 1 \}.

Prove that f is a one-to-one function and f(\mathbb{D}) = \{w : Re(w) > 0 \}.

2. The attempt at a solution

I've already shown the first part: Assume f(z_1) = f(z_2) for some z_1, z_2 \in \mathbb{C}, then z_1 = z_2. (I worked this out).

But for the second part, I'm not sure what to do. I've written the function in rectangular coordinates (z = x + iy) and the real part of the simplified fraction is:

\frac{1 - (x^2 + y^2)}{1 - 2y + x^2 + y^2}.

Now, I know that the numerator is nonnegative (since z \in \mathbb{D}, |z| < 1, so, x^2 + y^2 < 1). But, I am not certain about the sign of the denominator in the case where y > 0. Any ideas? And, if I can show this, will I have finished the proof, or do I have to show reverse inclusion?

Thanks in advance!
 
Physics news on Phys.org
This may be complex analysis, but do you still remember (y-1)^2=y^2-2y+1?
 
Man, it's always something really obvious. Thanks.
 
Question: A clock's minute hand has length 4 and its hour hand has length 3. What is the distance between the tips at the moment when it is increasing most rapidly?(Putnam Exam Question) Answer: Making assumption that both the hands moves at constant angular velocities, the answer is ## \sqrt{7} .## But don't you think this assumption is somewhat doubtful and wrong?

Similar threads

  • · Replies 17 ·
Replies
17
Views
3K
  • · Replies 19 ·
Replies
19
Views
4K
Replies
6
Views
5K
  • · Replies 6 ·
Replies
6
Views
2K
  • · Replies 1 ·
Replies
1
Views
1K
Replies
7
Views
2K
Replies
8
Views
3K
Replies
2
Views
1K
  • · Replies 27 ·
Replies
27
Views
2K
Replies
8
Views
2K